LSAT and Law School Admissions Forum

Get expert LSAT preparation and law school admissions advice from PowerScore Test Preparation.

 eli
  • Posts: 1
  • Joined: May 16, 2011
|
#22349
Section 1: Logical Reasoning

#23: What is the argument saying? Is there a way to answer the question without trying to fully understand what the convoluted argument is saying?
 Jon Denning
PowerScore Staff
  • PowerScore Staff
  • Posts: 904
  • Joined: Apr 11, 2011
|
#22350
#23: To answer this you need to see the flaw. The argument is essentially that since the statisticians are advising you to never add beliefs, but only to remove beliefs when they are proven false, your belief set will only shrink as you gain information. There are two problems with the argument: 1. The conclusion is that we need "many" beliefs, and, even with a shrinking belief set, one could still have "many" beliefs. So the statisticians' advice doesn't necessarily contradict "many." And 2. The advice is only about increasing the overall correctness of a belief set, not about survival. So the claim about how to increase correctness could still be true, even if it means that our survival is threatened by it. Answer choice A describes this second flaw: correctness and survival can be exclusive of one another.


I hope this helps!

JD
 martinbeslu
  • Posts: 49
  • Joined: Aug 09, 2017
|
#49926
Is it even possible to reject a belief without accepting a new belief? Wouldn't the very act of rejecting a specific belief entail believing that the belief is to be rejected?

The definition of belief is: an acceptance that a statement is true or that something exists.
The definition of reject is: dismiss as inadequate, inappropriate, or not to one's taste.

For example, if someone believes that all cars are red, then by rejecting this belief wouldn't they be creating a new belief that it's inadequate, inappropriate, or not to their taste to believe that all cars are red?

That is why I couldn't eliminate B. Is my reasoning incorrect or is B wrong for some other reason? I know we are looking for a flaw in the reasoning and B seems to point out this unreasonable assumption that you can reject a belief without forming a new belief.
 Adam Tyson
PowerScore Staff
  • PowerScore Staff
  • Posts: 5153
  • Joined: Apr 14, 2011
|
#50104
Your analysis certainly presents a conundrum, and gives us another reason to reject the statisticians' claim, but its the author's argument that we are supposed to be analyzing here, not that of the statisticians.

Answer B is a self-contradiction. How can we accept the statisticians' rule - never change one's beliefs except to reject some of them - and also accept new beliefs? Are we following his rule, or are we violating it? That's not a problem with the author's argument, but a problem with the statisticians' argument, and the author's flaw isn't that he didn't consider that, because if he did consider it he would be all the more certain of his conclusion.

Be sure that you are focusing on the author's argument, and why his reasoning isn't good enough to support his conclusion that the statisticians must be wrong.
User avatar
 simonsap
  • Posts: 34
  • Joined: Jun 14, 2021
|
#87951
It took a while to make sense of this one. Process of elimination is your best friend because A, C, D, and E are blatantly wrong, so you will get to B regardless. Now to justify B according to the wording of the passage :roll:

The key is in the phrase "total set of one's beliefs: never CHANGE THAT set, except by rejecting a belief WHEN given adequate evidence against it"

When you are given evidence against a belief in your set, you CHANGE the set. No where does it say you cannot ADD to it, only that you cannot MODIFY it without removing.
User avatar
 simonsap
  • Posts: 34
  • Joined: Jun 14, 2021
|
#87953
simonsap wrote: Tue Jun 15, 2021 9:36 am It took a while to make sense of this one. Process of elimination is your best friend because A, C, D, and E are blatantly wrong, so you will get to B regardless. Now to justify B according to the wording of the passage :roll:

The key is in the phrase "total set of one's beliefs: never CHANGE THAT set, except by rejecting a belief WHEN given adequate evidence against it"

When you are given evidence against a belief in your set, you CHANGE the set. No where does it say you cannot ADD to it, only that you cannot MODIFY it without removing.
The play is on semantics
User avatar
 PresidentLSAT
  • Posts: 87
  • Joined: Apr 19, 2021
|
#91601
Hello there, can a Powerscore team member help with my reasoning; this question was a nightmare.

Per some statisticians, the most secure way to increase the correctness of a belief set is keep it intact-except when you receive new information that discredit your existing set.

The author is citing that if we go with what these statisticians are saying, we have a set that keeps reducing.

Now wait, 2 possibilities were offered; we could also have an unchanged set or a reduction. Why is the author resorting to only one?

The author makes an unjustified connection to survival. Where did this connection pop up from? Per the author, the statisticians are mistaken because their cause of recommendation does not help with our survival.

In A, the choice captures the flaw in some word salad but it's essentially the best choice. A is saying that the author presumes (without justification) that ensuring the correctness of a set of beliefs must ensure our survival isn't in jeopardy. The "not" in this answer is what threw me off

B seemed attractive because after all, why can't I follow other rules in addition to what the statisticians are recommending? It seems to violate LSAT rules by leaning on outside information from the stimulus. We don't get to follow other rules because we are limited to the two we have been given. Extremely confusing. More help on the wrongness of this will be appreciated.

C. I don't know what C is saying but I ruled it out because the argument wasn't really about belief sizes and level of correctness.

D. This I also ruled out because the author did NOT advocate accepting believes on the basis of the strength of the evidence. The author discredited the statisticians beliefs because they did not guarantee survival. The "not" in A still confuses me.

E. I'm not sure if the argument assumes what E is saying. This is far from the conclusion and became a safe rule out for me.

Thanks in advance for the help
 Robert Carroll
PowerScore Staff
  • PowerScore Staff
  • Posts: 1774
  • Joined: Dec 06, 2013
|
#91681
simon,

Answer choice (A) is the correct answer, not answer choice (B).

PresidentLSAT,

It's true that the number of beliefs could stay the same, so wouldn't have to shrink. Further, "many" beliefs may represent a threshold still exceeded even as someone reduced their beliefs. So the author is making a mistake in that area, as Jon Denning pointed out earlier in this thread. If there's any answer choice that points out that flaw, I'd pick it.

There's another flaw, as already discussed, that the author assumes that beliefs needed to survive couldn't be wrong. We'll keep that in mind if we see it in an answer choice .

Answer choice (A) talks about that second flaw. The author says, essentially, "We need a lot of beliefs to survive, so correctness of beliefs can't depend on rejecting them." The statisticians' procedure tends to reduce beliefs, and the author thinks that survival requires a certain minimal set of beliefs. In response, I say: so what? Must it be that correct beliefs are good for survival? And that's what answer choice (A) is getting at.

Answer choice (B) cannot possibly be right - the statisticians' procedure explicitly disallows ever adding to one's beliefs. Who could follow their rule and add beliefs? That's not possible.

Answer choice (C) isn't a flaw the author commits at all.

Answer choice (D) you got rid of for a good reason.

Answer choice (E) is not something the author assumes at all.

Robert Carroll

Get the most out of your LSAT Prep Plus subscription.

Analyze and track your performance with our Testing and Analytics Package.